nlschultheis
Thanks Received: 0
Vinny Gambini
Vinny Gambini
 
Posts: 4
Joined: October 02nd, 2012
 
 
 

Q11 - Taken together some 2,000 stocks

by nlschultheis Wed Nov 28, 2012 7:05 pm

I had it between C and D for this and chose C.

I do not get how C weakens the argument, and if it does it seems to require outside info about the value of dividends vs share value, which I have very little of apparently.

I see how D more so strengthens the argument and is probably a better answer choice, but I thought Weaken EXCEPT answers did not have to strengthen, they could in fact be neutral, which is what C appears to me to be.
User avatar
 
ohthatpatrick
Thanks Received: 3808
Atticus Finch
Atticus Finch
 
Posts: 4661
Joined: April 01st, 2011
 
 
 

Re: Q11 - Taken together some 2,000 stocks

by ohthatpatrick Sat Dec 01, 2012 11:03 pm

You are correct about what you said regarding Weaken EXCEPT: the correct answer doesn't have to strengthen, it could merely be neutral.

But by your own account, you thought (D) strengthened the argument, while (C) felt more neutral.

I would suggest that if you ever have that feeling again, you go with the answer that is farther away from doing what you're looking for. (Strengthen is definitely farther from Weaken than Neutral is)

Naturally, (C) isn't really neutral, it does Weaken (they wouldn't make us pick Strengthen vs. Neutral on an EXCEPT question). But my point is just that since you understood that (D) would strengthen (and weren't too sure what (C) was saying), it's probably a stronger guess to go with (D).

We don't really need outside knowledge of "dividends" vs. "share value" to argue that (C) would weaken.

It suffices to see that "the experts were measured by Method X" while "the market was measured by Method Y".

If they were measured by two different methods, then is it really fair to compare their results?

(C) basically undermines the premise that the recommended stocks "performed less successfully" than the market as a whole. How can we say that the recommended stocks performed less successfully if their performance was calculated differently from the market's performance?

(D), meanwhile, does strengthen minutely by lending more credibility to the evidence. It essentially does the opposite of what (C) does. (D) makes us feel better that the observed difference in performance is accurate, since a number of analysts independently agreed with that claim.

Hope this helps.
 
jreeve12
Thanks Received: 0
Forum Guests
 
Posts: 4
Joined: May 08th, 2013
 
 
 

Re: Q11 - Taken together some 2,000 stocks

by jreeve12 Sat May 18, 2013 3:38 pm

I chose answer choice (E). Could you please tell me what I'm missing here?

The conclusion of the stimulus states "So clearly, no one should ever follow any recommendations by these experts"

Yet, answer choice (E) states that "the stock portfolios for which the guests were consultants performed better."

How does that added information weaken the argument that no one should listen to the expertswhen answer choice (E) doesn't even mention the advice of these same "successful consultants"??
 
sumukh09
Thanks Received: 139
Atticus Finch
Atticus Finch
 
Posts: 327
Joined: June 03rd, 2012
 
 
trophy
Most Thanked
trophy
First Responder
 

Re: Q11 - Taken together some 2,000 stocks

by sumukh09 Sat May 18, 2013 4:36 pm

I think E basically hits on the idea that these consultants know what they're doing so they are in fact in a position to be giving recommendations. If the consultants' portfolios performed better than the market as a whole, then maybe they know what they are talking about? It's too strong of a conclusion to say "no one should EVER" follow the recommendations of these consultants just because the recommendations they gave on this show didn't do better than the market in a single 12 year period.
 
alex.cheng.2012
Thanks Received: 8
Forum Guests
 
Posts: 28
Joined: May 02nd, 2013
 
 
 

Re: Q11 - Taken together some 2,000 stocks

by alex.cheng.2012 Thu Aug 01, 2013 3:59 am

I'd like to add a little bit to why answer (E) is wrong - why answer (E) weakens the argument.

Notice how the argument's conclusion isn't "no one should ever follow any advice given by these guests on the show." Rather, the argument's conclusion is "no one should ever follow any advice given by these guests/experts, period."

Answer (E) points out that these guests, despite their less than successful record in regards to the advice given on the show, are actually successful outside the show (when they're doing their real job as being consultants).

Additionally, the argument also says that most of these guests are successful consultants for multibillion dollar stock portfolios. Although what exactly "success" is isn't explicitly defined, this should still be a hint that answer (E) falls in line with the argument.
 
contropositive
Thanks Received: 1
Atticus Finch
Atticus Finch
 
Posts: 105
Joined: February 01st, 2015
 
 
 

Re: Q11 - Taken together some 2,000 stocks

by contropositive Wed Apr 08, 2015 2:45 am

Can someone explain B? I picked this answer choice because I thought it doesn't really do anything to the argument - weaken nor strengthen, which as I learned in my Manhatten LR 4th edition is what I am supposed to be looking for in the EXCEPT question types.

The argument core is:

the stocks recommended by the experts were less successful than the market as a whole for the 12-year period. THEREFORE, no one should take any recommendations from these so-called experts.

The reason (B) doesn't do anything to the argument is because the argument does not speak of "stock portfolios that were selected by other means." in fact, there is no mention of stock portfolios.
User avatar
 
ohthatpatrick
Thanks Received: 3808
Atticus Finch
Atticus Finch
 
Posts: 4661
Joined: April 01st, 2011
 
 
 

Re: Q11 - Taken together some 2,000 stocks

by ohthatpatrick Tue Apr 14, 2015 2:52 pm

Let me put up a complete explanation.

Question Type: Weaken EXCEPT

Argument Core:

C - No one should ever follow any recommendations by these so-called experts
why?
P - Over a 12 yr period, the 2000 stocks they recommended did worse than the market as a whole.

Evaluation
Whoa, that's an EXTREME conclusion! NO ONE should EVER follow ANY recommendations?

What if one of these so-called experts recommends that you step out of the crosswalk because a bus is speeding towards you?

In terms of the actual show / stocks / evidence ... just because the experts' stocks did worse than the overall market, does that mean the stocks did BADLY? Maybe those stocks still grew by 15% and people would have been happy with that growth rate.

Maybe these experts hid their REAL picks from the TV audience, because they like to only tell their REAL picks to their high priced clients. So if you're one of the high priced clients, then maybe YOU should follow the experts' recommendations.

I would look for answer choices that either give me
- a reason to think there MAY be situations when you WOULD want to listen to these experts
or
- a reason to think that the experts' 2000 stock predictions are not as unflattering as the premise wants to make them seem

(A) Maybe ... this says that on a different time scale, the experts' picks WERE superior. Generally, though, we measure the success of stock picks long term, so this answer seems a little weird.

(B) This definitely weakens. The experts' picks are the BEST AVAILABLE option! We should definitely take their recommendation.

(C) Oh, this undermines the force of the evidence, so it Weakens. In order for me to think the experts' picks were inferior to the overall market, I need an "apples to apples" comparison. If you're ranking them on different metrics, then the experts' picks might be better on one level, while the overall market is better on another level. That doesn't make either one superior/inferior. It just means "it depends what you're looking for". For people who are most concerned with dividends, maybe they SHOULD take the experts' recommendations.

(D) This strengthens the force of the evidence, so it Strengthens. Therefore, this will have to be the answer. This makes me trust the evidence more than before.

(E) Ooh, this is sort of like my "maybe they save their best picks for their high paying clients" guess. This Weakens by showing that although the TV audience might not want to follow the experts' TV recommendations, there might be other people who really SHOULD follow the experts' recommendations.

With both (C) and (D), you might be confused by answer choices that focus less on the truth value of the conclusion and more on the trusthworthiness / logical force of the evidence. Both of those are valid attack points for Strengthen / Weaken.

In an actual trial, we might have a bloody glove found in the defendant's car and covered in the victim's blood. Does this PROVE the conclusion that the defendant killed the victim? No.

We might try to introduce new evidence that sounds like our defendant couldn't possibly be the killer: he's too nice / he has no motive / he's too frail to carry out the vicious attack.

But we can also attack the credibility of glove itself: does it fit the defendant's hand? If it does, we're more inclined to believe it could have been worn by the defendant. If it doesn't, we're less inclined to believe that the defendant actually wore it.

Yes, I'm ripping off the O.J. Simpson trial. :) The point is that you can weaken arguments by attacking the plausibility of the conclusion AND by attacking the trustworthiness of the evidence.

Hope this helps.